Đến nội dung

Hình ảnh

Số học -Tuyển tập các bài toán sưu tầm từ Mathlinks.ro

- - - - - tuyển tập-sưu tầm.

  • Please log in to reply
Chủ đề này có 44 trả lời

#21
hxthanh

hxthanh

    Tín đồ $\sum$

  • Hiệp sỹ
  • 3903 Bài viết

Spoiler

Bài 13

Rõ ràng $\lfloor a\rfloor=1$ không phải là nghiệm, nên phương trình đã cho tương đương với:

$\left\lfloor\dfrac{a}{1-a}\right\rfloor=\dfrac{\lfloor a\rfloor}{1-\lfloor a\rfloor}=m\in\mathbb Z$

Suy ra:

$1-\lfloor a\rfloor \;\bigg|\; \lfloor a\rfloor\Rightarrow 1-\lfloor a\rfloor \;\bigg|\; 1\Rightarrow 1-\lfloor a\rfloor=\pm 1$

$\Rightarrow \left[\begin{align*}\lfloor a\rfloor=0\\ \lfloor a\rfloor=2\end{align*}\right.$

Như vậy phương trình đã cho sẽ tương đương với:

$\left[\begin{align*}&\begin{cases}\left\lfloor\dfrac{a}{1-a}\right\rfloor=0\\ 0\le a<1\end{cases}\\ & \\ &\begin{cases}\left\lfloor\dfrac{a}{1-a}\right\rfloor=-2\\ 2\le a<3\end{cases}\end{align*}\right.$ $\Rightarrow \left[ \begin{align*} 0\le a<\frac{1}{2} \\ 2\le a<3\end{align*}\right.$

 

@Dark templar: Mấy bài này em cho ra để các bạn biết dạng căn bản giải thôi mà anh. :P


Bài viết đã được chỉnh sửa nội dung bởi hxthanh: 21-04-2013 - 20:20
Đã sửa


#22
hxthanh

hxthanh

    Tín đồ $\sum$

  • Hiệp sỹ
  • 3903 Bài viết

Bài toán 12: Giải PT $\left\lfloor {{x^2} + 1} \right\rfloor  = \left\lfloor {2x} \right\rfloor $.

Phương trình đã cho tương đương với:

$\lfloor x^2+1-\lfloor 2x\rfloor \rfloor=0$

$\Rightarrow 0\le x^2+1-\lfloor 2x\rfloor <1\Rightarrow x^2<\lfloor 2x\rfloor <2x\Rightarrow 0<x<2$

$\Rightarrow \lfloor 2x\rfloor \in\{0,1,2,3\}$

Thay trực tiếp ta có:

$\begin{cases}\lfloor 2x\rfloor=0\\ \lfloor x^2+1\rfloor=0\end{cases}\quad\Rightarrow\quad \text{vô nghiệm}$

$\begin{cases}\lfloor 2x\rfloor=1\\ \lfloor x^2+1\rfloor=1\end{cases}\quad\Rightarrow\quad \dfrac{1}{2}\le x<1$

$\begin{cases}\lfloor 2x\rfloor=2\\ \lfloor x^2+1\rfloor=2\end{cases}\quad\Rightarrow\quad 1\le x<\sqrt 2$

$\begin{cases}\lfloor 2x\rfloor=3\\ \lfloor x^2+1\rfloor=3\end{cases}\quad\Rightarrow\quad \dfrac{3}{2}\le x<\sqrt 3$

 

Vậy nghiệm phương trình đã cho là:

$\boxed{\;x\in\left[\frac{1}{2},\;\sqrt 2\right)\cup\left[\frac{3}{2},\;\sqrt 3\right)\;}$



#23
dark templar

dark templar

    Kael-Invoker

  • Hiệp sỹ
  • 3788 Bài viết


Spoiler

Bài 13

Rõ ràng $\lfloor a\rfloor=1$ không phải là nghiệm, nên phương trình đã cho tương đương với:

$\left\lfloor\dfrac{a}{1-a}\right\rfloor=\dfrac{\lfloor a\rfloor}{1-\lfloor a\rfloor}=m\in\mathbb Z$

Suy ra:

$1-\lfloor a\rfloor \;\bigg|\; \lfloor a\rfloor\Rightarrow 1-\lfloor a\rfloor \;\bigg|\; 1\Rightarrow 1-\lfloor a\rfloor=\pm 1$

$\Rightarrow \left[\begin{align*}\lfloor a\rfloor=0\\ \lfloor a\rfloor=2\end{align*}\right.$

Như vậy phương trình đã cho sẽ tương đương với:

$\left[\begin{align*}\begin{cases}\left\lfloor\dfrac{a}{1-a}\right\rfloor=0\\ 0\le a<1\end{cases}\\ \\ \begin{cases}\left\lfloor\dfrac{a}{1-a}\right\rfloor=2\\ 2\le a<3\end{cases}\end{align*}\right.\Rightarrow \boxed{\displaystyle 0\le a<\frac{1}{2}}$

Bài này anh còn thiếu 1 tập nghiệm nữa là $a \in [2;3)$,lý do là khi thay $\left\lfloor a \right\rfloor =2$ thì phải ra hệ là $\left\{ \begin{array}{l}\left\lfloor {\frac{a}{{1 - a}}} \right\rfloor  =  - 2\\2 \le a < 3\end{array} \right. \Rightarrow 2 \le a < 3$.

 

Từ đó tập nghiệm của PT là $\boxed{\displaystyle a \in \left[ {0,\frac{1}{2}} \right) \cup [2,3)}$.

 

**********

Đề mới:

 

Bài toán 14: Dãy ${a_1},{a_2},{a_3},...$ thỏa mãn ${a_{4n + 1}} = 1,{a_{4n + 3}} = 0,{a_{2n}} = {a_n}$.Chứng minh dãy này là dãy không tuần hoàn.

 

Bài toán 15: Tìm số nguyên $N$ lớn nhất thỏa mãn:

  1. $\left\lfloor {\frac{N}{3}} \right\rfloor $ có 3 chữ số giống nhau.
  2. $\left\lfloor {\frac{N}{3}} \right\rfloor  = 1 + 2 + 3 +  \cdots  + n$ với $n$ là số nguyên dương nào đó.

     


"Do you still... believe in me ?" Sarah Kerrigan asked Jim Raynor - Starcraft II:Heart Of The Swarm.

#24
hxthanh

hxthanh

    Tín đồ $\sum$

  • Hiệp sỹ
  • 3903 Bài viết
Bài toán 15: Tìm số nguyên $N$ lớn nhất thỏa mãn:
  1. $\left\lfloor {\frac{N}{3}} \right\rfloor $ có 3 chữ số giống nhau.
  2. $\left\lfloor {\frac{N}{3}} \right\rfloor  = 1 + 2 + 3 +  \cdots  + n$ với $n$ là số nguyên dương nào đó.

Bài 15 này lại mang tính chất "lừa tình"

Ta có:

$1+2+...+n=\frac{n(n+1)}{2}=a.111$ với $1\le a\le 9$

Suy ra:

$n(n+1)=2.3.37.a$

Nhận thấy rằng VT là tích của 2 số tự nhiên liên tiếp (trong đó có 1 số chẵn và một số lẻ)

Số chẵn không thể là $2a$ vì $2a\le 18<3.37-1$

cũng vậy số chẵn đó không thể là $2.37.a$ vì $2.37.a>4$

Do đó số chẵn đó là $6a$ số lẻ là $37$ với $1\le a\le 9$ Dễ thấy $a=6$ thỏa mãn $\Rightarrow 1+2+...+n=666$

$\Rightarrow \left\lfloor\dfrac{N}{3}\right\rfloor=666\Rightarrow 666\le \frac{N}{3}<667\Rightarrow 1998\le N<2001$

 

Do đó $\boxed{N_{\max}=2000}$



#25
dark templar

dark templar

    Kael-Invoker

  • Hiệp sỹ
  • 3788 Bài viết

Bài toán 14: Dãy ${a_1},{a_2},{a_3},...$ thỏa mãn ${a_{4n + 1}} = 1,{a_{4n + 3}} = 0,{a_{2n}} = {a_n}$.Chứng minh dãy này là dãy không tuần hoàn.

Lời giải bài toán 14:

Giả sử ${a_{n + p}} = {a_n},\forall n$ và với $p>0$ nào đó.

 

Đặt $p = {2^a}q$ với $q$ lẻ.

 

 

Cho $n = {2^{a + 2}}(q + 2)$

 

Sử dụng tính tuần hoàn,ta có ${a_{n + p}} = {a_n} = {a_{q + 2}}$,nhưng ${a_{n + p}} = {a_{{2^a}(q + 4(q + 2))}} = {a_{q + 4(q + 2)}}$

 

Mặt khác $q+2$ và $q+4(q+2)$ là 2 số lẻ khác nhau theo $\mod 4$.

 

Vậy ${a_{q + 2}} \ne {a_{q + 4(q + 2)}}$ (một là 0;hoặc là 1).

 

Vậy không có $p>0$ nào tồn tại và do đó dãy không tuần hoàn.

 

**********

Đề mới:

 

Bài toán 16: Tìm số tự nhiên $n = \overline {abc} $ thỏa mãn đồng thời ${n^{69}} = \overline {1986...} $ và ${n^{121}} = \overline {3333...} $

 

Bài toán 17: Tính $\sum\limits_{i = 1}^m {\left\lfloor {\sqrt {2i - 1} } \right\rfloor } $


"Do you still... believe in me ?" Sarah Kerrigan asked Jim Raynor - Starcraft II:Heart Of The Swarm.

#26
dark templar

dark templar

    Kael-Invoker

  • Hiệp sỹ
  • 3788 Bài viết

Lời giải cho bài 16,17 sẽ post sau.Đề mới:

 

Bài toán 18: Cho $n \ge 2$ và $a_{n}x+b_{n}$ là số dư khi chia $x^{n}$ cho $x^2-6x-12$.Với mỗi $n$,hãy tìm tất cả các số nguyên tố là ước chung của $a_{n}$ và $b_{n}$.

 

Bài toán 19: Giải PT nghiệm nguyên dương $\left\lfloor {\frac{{{x^2}}}{y}} \right\rfloor  + \left\lfloor {\frac{{{y^2}}}{x}} \right\rfloor  = \left\lfloor {\frac{x}{y} + \frac{y}{x} + xy} \right\rfloor $.

 


Bài viết đã được chỉnh sửa nội dung bởi dark templar: 03-06-2013 - 22:09

"Do you still... believe in me ?" Sarah Kerrigan asked Jim Raynor - Starcraft II:Heart Of The Swarm.

#27
dark templar

dark templar

    Kael-Invoker

  • Hiệp sỹ
  • 3788 Bài viết


Đề mới:

 

Bài toán 16: Tìm số tự nhiên $n = \overline {abc} $ thỏa mãn đồng thời ${n^{69}} = \overline {1986...} $ và ${n^{121}} = \overline {3333...} $

 

Bài toán 17: Tính $\sum\limits_{i = 1}^m {\left\lfloor {\sqrt {2i - 1} } \right\rfloor } $

Lời giải bài toán 16:

Đặt $a=\log_{10}n \in [2;3)$.

 

Gọi $p$ là số các chữ số $1;9;8;6$ viết trong hệ thập phân của $n^{69}$.

 

Gọi $q$ là số các chữ số $3;3;3;3$ viết trong hệ thập phân của $n^{121}$.

 

Ta có 2 đánh giá sau:

$p+\log_{10}1987>69a \ge p+\log_{10}1986$

$q+\log_{10}3334>121a \ge q+\log_{10}3333$.

 

$121p + 121{\log _{10}}(1987) > 69 \times 121a \ge 121p + 121{\log _{10}}(1986)$

$69q + 69{\log _{10}}(3334) > 69 \times 121a \ge 69q + 69{\log _{10}}(3333)$

 

$121p + 399.0... > 69 \times 121a \ge 121p + 399.0...$

$69q + 243.0... > 69 \times 121a \ge 69q + 243.0...$

 

Do đó $121p + 399 = 69q + 243$$ \Leftrightarrow 69q - 121p = 156$,như vậy thì:

$p = 66 + 69m$,$q = 118 + 121m$ và $[69 \times 121a] = 8385 + 8349m$

 

Vì $a \in [2;3)$ nên ta tìm được $m=1$ và $(p,q) = (135,239)$

 

$p + {\log _{10}}(1987) > 69a \ge p + {\log _{10}}(1986) \Rightarrow 2.00431853977043... \le a < 2.00432170821898...$

$q + {\log _{10}}(3334) > 121a \ge q + {\log _{10}}(3333) \Rightarrow 2.00432095300546... \le a < 2.00432202971481...$

 

Vậy $2.00432095300546... \le a < 2.00432170821898...$

$100.999902143645... \le n < 101.000077776911...$

 

Từ đó ta tìm được $\boxed{n=101}$ là số thỏa mãn yêu cầu bài toán.

 

Lời giải bài toán 17:

Với $\frac{{{k^2} + 1}}{2} \le i \le \frac{{{{(k + 1)}^2}}}{2}$ thì $\left\lfloor {\sqrt {2i - 1} } \right\rfloor  = k$

 

Vậy $\sum\limits_{i = 1}^{\left\lfloor {\frac{{{{(n + 1)}^2}}}{2}} \right\rfloor } {\left\lfloor {\sqrt {2i - 1} } \right\rfloor }  = \sum\limits_{k = 1}^n k \left( {1 + \left\lfloor {\frac{{{{(k + 1)}^2}}}{2}} \right\rfloor  - \left\lceil {\frac{{{k^2} + 1}}{2}} \right\rceil } \right)$

$ = \sum\limits_{k = 1,k{\text{ chẵn}}}^n k \left( {1 + \frac{{{{(k + 1)}^2} - 1}}{2} - \frac{{{k^2} + 2}}{2}} \right) + \sum\limits_{k = 1,k{\text{ lẻ}}}^n k \left( {1 + \frac{{{{(k + 1)}^2}}}{2} - \frac{{{k^2} + 1}}{2}} \right)$

$ = \sum\limits_{k = 1,k{\text{ chẵn}}}^n {{k^2}}  + \sum\limits_{k = 1,k{\text{ lẻ}}}^n k (k + 1) = \sum\limits_{k = 1,k}^n {{k^2}}  + \sum\limits_{k = 1,k{\text{ lẻ}}}^n k $

$ = \frac{{n(n + 1)(2n + 1)}}{6} + {\left\lfloor {\frac{{n + 1}}{2}} \right\rfloor ^2}$

 

Như vậy ta chọn $n$ sao cho $\left\lfloor {\frac{{{{(n + 1)}^2}}}{2}} \right\rfloor  \le m$,ta được:

$\sum\limits_{i = 1}^m {\left\lfloor {\sqrt {2i - 1} } \right\rfloor }  = \frac{{n(n + 1)(2n + 1)}}{6} + {\left\lfloor {\frac{{n + 1}}{2}} \right\rfloor ^2} + (n + 1)\left( {m - \left\lfloor {\frac{{{{(n + 1)}^2}}}{2}} \right\rfloor } \right)$

 

Từ BĐT $\left\lfloor {\frac{{{{(n + 1)}^2}}}{2}} \right\rfloor  \le m < \left\lfloor {\frac{{{{(n + 2)}^2}}}{2}} \right\rfloor $,ta có $n = \left\lfloor {\sqrt {2m + 1} } \right\rfloor  - 1$.

 

Như vậy:

$\sum\limits_{i = 1}^m {\left\lfloor {\sqrt {2i - 1} } \right\rfloor }  = km + \frac{{k(k - 1)(2k - 1)}}{6} + {\left\lfloor {\frac{k}{2}} \right\rfloor ^2} - k\left\lfloor {\frac{{{k^2}}}{2}} \right\rfloor {\text{    với   }}k = \left\lfloor {\sqrt {2m + 1} } \right\rfloor $

 

Hay có thể rút gọn là :

$\boxed{\displaystyle \sum\limits_{i = 1}^m {\left\lfloor {\sqrt {2i - 1} } \right\rfloor }  = km - \left\lfloor {\frac{{k(k + 2)(2k - 1)}}{{12}}} \right\rfloor {\rm{    với   }}k = \left\lfloor {\sqrt {2m + 1} } \right\rfloor }$

 



Bài toán 18: Cho $n \ge 2$ và $a_{n}x+b_{n}$ là số dư khi chia $x^{n}$ cho $x^2-6x-12$.Với mỗi $n$,hãy tìm tất cả các số nguyên tố là ước chung của $a_{n}$ và $b_{n}$.

 

Bài toán 19: Giải PT nghiệm nguyên dương $\left\lfloor {\frac{{{x^2}}}{y}} \right\rfloor  + \left\lfloor {\frac{{{y^2}}}{x}} \right\rfloor  = \left\lfloor {\frac{x}{y} + \frac{y}{x} + xy} \right\rfloor $.

Lời giải bài toán 18:

Ta có:

$ (3+\sqrt{21})a_n+b_n = (3+\sqrt{21})^n $

$ (3-\sqrt{21})a_n+b_n = (3-\sqrt{21})^n $

 

Suy ra:

$ a_n =\frac{(3+\sqrt{21})^n-(3-\sqrt{21})^n}{2\sqrt{21}}, b_n =\frac{(3+\sqrt{21})(3-\sqrt{21})^n-(3-\sqrt{21})(3+\sqrt{21})^n}{2\sqrt{21}} $

$ a_{n+2}= 6a_{n+1}+12a_n $

$ b_{n+2}= 6b_{n+1}+12b_n $

$ a_2 = 6, b_2 = 12 $,do đó $ ((a_2,b_2),(a_3,b_3),. . .,(a_n,b_n),...) = 6 $

 

Lời giải bài toán 19:

Đổi ký hiệu $(x;y)$ sang $(p;q)$.

 

Không mất tính tổng quát,giả sử $p \ge q$.

 

$(\star )q=1$,suy ra $ p^2+\left\lfloor\frac{1}{p}\right\rfloor $$ =2p+\left\lfloor\frac{1}{p}\right\rfloor $,suy ra $p=2$ và ta có 2 nghiệm đầu tiên là $(p;q)=(1;2);(2;1)$.

 

$(\star\star )q>1$.Từ PT ban đầu,ta có thể suy ra 2 BĐT sau:

$ \frac{p^2}{q}+\frac{q^2}{p}>\frac{p}{q}+\frac{q}{p}+pq-1 $

$ \frac{p^2}{q}+\frac{q^2}{p}-2<\frac{p}{q}+\frac{q}{p}+pq $

 

Hay:

$ P_1(p)=p^3-p^2(q^2+1)+pq+q^3-q^2>0 $

$ P_2(p)=p^3-p^2(q^2+1)-2pq+q^3-q^2<0 $

 

$ P_1(0)=q^3-q^2>0 $

$ P_1(q)=-q^4+2q^3-q^2<0 $

$ P_1(q^2)=-q^4+2q^3-q^2<0 $

$ P_1(q^2+1)=2q^3-q^2+q>0 $

 

Suy ra $ P_1(p)>0 $ và $p \ge q \iff p \ge q^2+1$.

 

$ P_2(0)=q^3-q^2>0 $

$ P_2(q^2+1)=-q^3-q^2-2q<0 $

 

Suy ra $P_2(x)$ có 3 nghiệm là $ r_1<0<r_2<q^2+1<r_3 $ và ta thu được $ r_3>p\ge q^2+1 $

 

Từ đó ta đặt $ p=q^2+a $ với $a \in \mathbb{N}$.PT $P_2(q)<0$ trở thành:

$ (q^2+a)^3-(q^2+a)^2(q^2+1)-2q(q^2+a)+q^3-q^2<0 $

 

Hay:

$ q^6+3q^4a+3q^2a^2+a^3-q^6-q^4-2aq^4-2aq^2-a^2q^2-a^2-2q^3-2aq+q^3-q^2<0 $

$ (a-1)q^4-q^3+q^2(2a^2-2a-1)+a^3-a^2-2aq<0 $

 

Dễ dàng chỉ ra rằng BĐT trên sai với $a \ge 2$(nhớ rằng $q>1$)

 

Do đó ta còn lại trường hợp là $p=q^2+1$.

 

Thay vào PT ban đầu,ta có thể thấy rằng đây chính là nghiệm (nhớ rằng $q>1$).

 

Như vậy kết quả cuối cùng là $ \boxed{x\in\{\sqrt{y-1},y^2+1\}\cap\mathbb{N}} $


"Do you still... believe in me ?" Sarah Kerrigan asked Jim Raynor - Starcraft II:Heart Of The Swarm.

#28
dark templar

dark templar

    Kael-Invoker

  • Hiệp sỹ
  • 3788 Bài viết

Đề mới:

 

Bài toán 20: Cho $a,b,c$ lần lượt là giá trị của phần trăm,phần mười và phần đơn vị của giá trị tổng $ \left(\sum_{n=1}^{1000}[(5n)!+(6n)!] \right)^2 $.Hãy tính $\frac{a^3+b^3+c^3}{a+b+c}$.

 

Bài toán 21: Chứng minh không tồn tại các số nguyên dương $n$ và $k$ thỏa mãn $ \lfloor(2+\sqrt 3)^{2n+1}\rfloor=\lfloor(4+\sqrt{15})^k\rfloor $


Bài viết đã được chỉnh sửa nội dung bởi dark templar: 03-06-2013 - 22:18

"Do you still... believe in me ?" Sarah Kerrigan asked Jim Raynor - Starcraft II:Heart Of The Swarm.

#29
hxthanh

hxthanh

    Tín đồ $\sum$

  • Hiệp sỹ
  • 3903 Bài viết
...

Lời giải bài toán 19:

Đổi ký hiệu $(x;y)$ sang $(p;q)$.

...

Như vậy kết quả cuối cùng là $ \boxed{x\in\{\sqrt{y-1},y^2+1\}\cap\mathbb{N}} $

Cách viết kết quả như thế này quả thực không "chuẩn" lắm!

Nên viết là:

$\begin{cases} x=u \\y=v \\ \sqrt{v-1}\le u\le v^2+1\\ u,v \in \mathbb N^*\end{cases}$



#30
hxthanh

hxthanh

    Tín đồ $\sum$

  • Hiệp sỹ
  • 3903 Bài viết

Bài toán 21: Chứng minh không tồn tại các số nguyên dương $n$ và $k$ thỏa mãn $ \lfloor(2+\sqrt 3)^{2n+1}\rfloor=\lfloor(4+\sqrt{15})^k\rfloor $

Có thể giải quyết bài này như sau:

Sử dụng tính chất phần nguyên: $\begin{cases}x,y\notin \mathbb Z\\ x+y\in \mathbb Z\end{cases} \Rightarrow \lfloor x\rfloor+\lfloor y\rfloor=x+y-1$

Ta có:

$\left\lfloor (2+\sqrt 3)^{2n+1}\right\rfloor+\left\lfloor (2-\sqrt 3)^{2n+1}\right\rfloor=(2+\sqrt 3)^{2n+1}+(2-\sqrt 3)^{2n+1}-1$

$\Rightarrow \left\lfloor (2+\sqrt 3)^{2n+1}\right\rfloor = (2+\sqrt 3)^{2n+1}+(2-\sqrt 3)^{2n+1}-1$

(Do $0<(2-\sqrt 3)^{2n+1}<1$)

Tương tự ta cũng có:

$\Rightarrow \left\lfloor (4+\sqrt{15})^{k}\right\rfloor = (4+\sqrt{15})^{k}+(4-\sqrt{15})^{k}-1$

Đặt

$\{A_n\}= \left\{(a_n)\;\Big|n\in \mathbb N^* \;;\; a_n=(2+\sqrt 3)^{2n+1}+(2-\sqrt 3)^{2n+1}\right\}$

$\{B_k\}= \left\{(b_k)\;\Big|k\in \mathbb N^* \;;\; b_k=(4+\sqrt{15})^k+(4-\sqrt{15})^k\right\}$

Như vậy điều cần chứng minh tương đương với:

$\{A_n\}\ne \{B_k\}$

Thật vậy, từ các công thức tổng quát ta lập lại biểu thức truy hồi.

$\{A_n\}=\left\{(a_n)\;\left|\;\begin{cases}a_1=52,\; a_2=724\\ a_{n+2}=14a_{n+1}-a_n\end{cases}\right.\right\}$

$\{B_k\}=\left\{(b_k)\;\left|\;\begin{cases}b_1=8,\; b_2=62\\ b_{k+2}=8b_{k+1}-b_k\end{cases}\right.\right\}$

Cách xác lập

Từ kết quả này ta xét số dư $A_n, B_k$ theo modulo $3$, và dễ dàng nhận ra: (chứng minh đơn giản bằng quy nạp)

$A_n\equiv 1\pmod 3\quad\forall n\in\mathbb N^*$

$B_k\equiv -1\pmod 3\quad\forall k\in\mathbb N^*$

Từ đây ta có điều phải chứng minh.

 

Hệ quả



#31
dark templar

dark templar

    Kael-Invoker

  • Hiệp sỹ
  • 3788 Bài viết

Đề mới:

 

Bài toán 22: Giải PT nghiệm nguyên dương $ (2^a-1)(3^b-1)=c ! $

 


 


"Do you still... believe in me ?" Sarah Kerrigan asked Jim Raynor - Starcraft II:Heart Of The Swarm.

#32
dark templar

dark templar

    Kael-Invoker

  • Hiệp sỹ
  • 3788 Bài viết


Bài toán 22: Giải PT nghiệm nguyên dương $ (2^a-1)(3^b-1)=c ! $

Spoiler

Lời giải bài toán 22:

Nếu $c \le 3$ thì ta có thể lập tức ta tìm ra nghiệm là $ (a,b,c)\in\left\{(1,1,2),(2,1,3)\right\} $.Nếu $c \ge 4$ thì $ v_2(c!)\geq\frac{c}{2} $ và $ v_2(c!)\geq 3 $.

 

Do $ 2\nmid2^a-1 $ ,ta có $ 2^{\left\lceil\frac{c}{2}\right\rceil}\mid 3^b-1 $ nếu $ (2^a-1)(3^b-1)=c! $.Nhưng để có $ 3^b\equiv 1\pmod{2^{\left\lceil\frac{c}{2}\right\rceil}} $,ta phải có $ \lambda\left(2^{\left\lceil\frac{c}{2}\right\rceil}\right)\mid b $ với $\lambda$ là hàm Carmichael. (các bạn có thể tham khảo về hàm số này ở đây.)

 

Ta có $ \lambda\left(2^{\left\lceil\frac{c}{2}\right\rceil}\right)=2^{\left\lceil\frac{c}{2}\right\rceil-2} $ và ta thu được $ c!=(2^a-1)(3^b-1)\geq 3^b-1\geq 3^{2^{\lceil c/2\rceil-2}} $.Thế nhưng điều này sai với $c \ge 13$(có thể chứng minh dễ dàng,chẳng hạn bằng quy nạp).

 

Ta kiểm tra $ c\in\left\{4,5,\dots,12\right\} $ sử dụng $ v_2(c!) $ và tìm được $ (a,b,c)\in\left\{(2,2,4),(4,2,5),(6,4,7)\right\} $.Hoặc ta có thể hoàn thiện các đánh giá của ta cho $v_2(c!)$ với $c \ge 8$.

 

Vậy ta có các nghiệm:

\[ \boxed{(a,b,c)\in\left\{(1,1,2),(2,1,3),(2,2,4),(4,2,5),(6,4,7)\right\}} \]

 

====================

Đề mới:

 

Bài toán 23: Tìm tất cả số nguyên $a,b,n>1$ thỏa $(a^3+b^3)^{n}=4(ab)^{1995}$

 

Bài toán 24: Có bao nhiêu nghiệm nguyên của PT $|x|+|y|+|z|=n$ ,nếu ta giả sử các bộ hoán vị của nghiệm $(x_0;y_0;z_0)$ được tính là các nghiệm khác nhau ?

 

Bài toán 25: Chứng minh rằng tồn tại vô số số nguyên dương $n$ sao cho các số $\binom{n}{0};\binom{n}{1};...;\binom{n}{n}$ đều là số lẻ.

-43-


Bài viết đã được chỉnh sửa nội dung bởi dark templar: 10-06-2013 - 21:43

"Do you still... believe in me ?" Sarah Kerrigan asked Jim Raynor - Starcraft II:Heart Of The Swarm.

#33
barcavodich

barcavodich

    Sĩ quan

  • Thành viên
  • 449 Bài viết

Giải $22$ trước vậy

Đầu tiên đặt $gcd(a,b)=d$

$\Rightarrow a=dx,b=dy$ với $(x,y)=1$

Từ giả thiết ta có

$d^{3n}.(x^3+y^3)^n=4d^{3990}.(xy)^{1995}$

Lại do $gcd(x,y)=gcd(x^3+y^3,x)=gcd(x^3+y^3,y)=1$

Nên $x=y=1$

$\Rightarrow a=b$ thay vào ta được

$ 2^na^{3n}= 4a^{3990} $

Hay $ 2^{n-2}= a^{3990-3n} $

Do đó $a$ là lũy thừa của $2$ 

Đặt $ a = 2^q $ ta có $ 2^{n-2}= 2^{q(3990-3n)} $

$\Rightarrow n-2 = q(3990-3n)$

$\Rightarrow$$ n =\frac{3990q+2}{3q+1}=\frac{3990q+1330-1328}{3q+1}= 1330-\frac{1328}{3q+1} $

Do $n$ nguyên nên $3q+1|1330$

Từ đó tìm được các nghiệm $(a,b,n)$ sau

$\boxed{(1,1,2);(2, 2, 998); (32, 32, 1247); (2^{55}, 2^{55}, 1322); (2^{221}, 2^{221}, 1328)}$


Bài viết đã được chỉnh sửa nội dung bởi barcavodich: 10-06-2013 - 22:46

[topic2=''][/topic2]Music makes life more meaningful


#34
dark templar

dark templar

    Kael-Invoker

  • Hiệp sỹ
  • 3788 Bài viết

Bài toán 26: Cho $a,b,c,d \in \mathbb{R}$ thỏa mãn $\forall n \in \mathbb{N}:an^3+bn^2+cn+d$ là số nguyên.Chứng minh $6a;6b;6c;6d$ là các số nguyên.

-46-


"Do you still... believe in me ?" Sarah Kerrigan asked Jim Raynor - Starcraft II:Heart Of The Swarm.

#35
hxthanh

hxthanh

    Tín đồ $\sum$

  • Hiệp sỹ
  • 3903 Bài viết

Bài toán 17: Tính $S_m=\sum\limits_{i = 1}^m {\left\lfloor {\sqrt {2i - 1} } \right\rfloor } $

Bài này nếu đặt ${}\;\;n=\left\lfloor\sqrt{2m-1}\right\rfloor\;\;{}$ thì sau khi tính toán ta được kết quả là:

 

$S_m=nm-{n+1\choose 3}-\left\lfloor\frac{n^2}{4}\right\rfloor$

 

Gợi ý



#36
hxthanh

hxthanh

    Tín đồ $\sum$

  • Hiệp sỹ
  • 3903 Bài viết

Bài toán 24: Có bao nhiêu nghiệm nguyên của PT $|x|+|y|+|z|=n$ ,nếu ta giả sử các bộ hoán vị của nghiệm $(x_0;y_0;z_0)$ được tính là các nghiệm khác nhau ?

 

Bài này có thể làm như sau:

 

Với $n=0$ hiển nhiên phương trình có nghiệm duy nhất $(x,y,z)=(0,0,0)$

Với $n>0$

$\boxed{\textbf{Case 1:}}$

Xét bộ nghiệm có dạng $(0,b,c)$ với $b\ne 0\ne c$ (và tương tự cho $(a,0,c)$ và $(a,b,0)$)

Với mỗi bộ $(0,b,c)$ thì tương ứng có $2^2=4$ bộ nghiệm thỏa mãn (là $(0,b,c);\;(0,-b,c);\;(0,b,-c);\;(0,-b,-c)$) nên ta chỉ cần đếm số bộ nghiệm nguyên dương của phương trình ${}\quad b+c=n$

Số nghiệm này bằng ${n-1\choose 1}=n-1$ (Bài toán chia kẹo Euler)

 

Như vậy các trường hợp 1, cho ta $4.(n-1).3$ bộ nghiệm.

 

$\boxed{\textbf{Case 2:}}$

Xét bộ nghiệm có dạng $(0,0,c)$ với $c\ne 0$ (và tương tự cho $(0,b,0)$ và $(a,0,0)$)

 

Các trường hợp 2 này sẽ cho ta $2^1.1.3=6$ bộ nghiệm.

 

$\boxed{\textbf{Case 3:}}$

Bây giờ chỉ phải xét số bộ nghiệm nguyên dương của phương trình ${}\quad a+b+c=n$ (rồi nhân với $2^3$)

Kết quả (bài toán kẹo Euler) này là: ${n-1\choose 2}$

 

Result: Số nghiệm nguyên của phương trình đã cho là:

 

$S_n=\begin{cases}1&{}\quad n=0\\ 12(n-1)+6+4(n-1)(n-2)=\boxed{4n^2+2}&{}\quad n>0\end{cases}$

 

Hay viết một cách thu gọn lại thì $$S_n=4n^2+2-\left\lfloor\frac{1}{n+1}\right\rfloor$$



#37
nguyenthehoan

nguyenthehoan

    Sĩ quan

  • Thành viên
  • 392 Bài viết

[

Bài toán 25: Chứng minh rằng tồn tại vô số số nguyên dương $n$ sao cho các số $\binom{n}{0};\binom{n}{1};...;\binom{n}{n}$ đều là số lẻ.

-43-

Ta phát biểu và chứng minh bổ đề sau:Định lí Lucas:

 

Với mọi $0\leq k\leq 2^{n}-1$ thì $\binom{2^{n}-1}{k}$ là số lẻ.

 

Chứng minh:

 

ta có $\binom{2^{n}-1}{k}=\frac{(2^{n}-1)!}{k!(2^{n}-1-k)!}$

 

Ta sẽ chứng minh $v_{2}((2^{n}-1)!)=v_{2}(k!)+v_{2}((2^{n}-1-k)!)$

 

Thật vậy.Ta kí hiệu $s_{k}(n)$ là tổng các chữ số của $n$ khi viết trong hệ cơ số $k$.

 

Ta có $v_{2}(n)=n-s_{2}(n)$ (quen thuộc)

 

Từ đó ta phải cm $s_{2}(2^{n}-1)=s_{2}(k)+s_{2}(2^{n}-1-k)$

 

Đặt $k=a_{0}+a_{1}.2+...+a_{m}2^{m}$ với $a_{i}$ bằng $0$ hoặc $1$)$\Rightarrow s_{2}(k)=a_{0}+a_{1}+..+a_{m}$

 

Và $2^{n}-1-k=(1-a_{0})+(1-a_{1}).2+...+(1-a_{m}).2^{m}+2^{m+1}+...+2^{n-1}$

 

$\Rightarrow s_{2}(2^{n}-1-k)=s_{2}(2^{n}-1)-s_{2}(k)$ (bổ đề được cm)

 

Áp dụng vào bài toán ta có ngay DPCM.



#38
WhjteShadow

WhjteShadow

    Thượng úy

  • Phó Quản lý Toán Ứng dụ
  • 1323 Bài viết

Bài toán 26 :

Ta sử dụng 1 bổ đề cơ bản tr0ng đa thức : Mọi đa thức bậc $n$ đều có thể được biểu diễn dưới dạng :

$$P_{n}(x)=b_0+b_1.\binom{x}{1}+b_2.\binom{n}{2}+.....+b_{n}.\binom{x}{n}$$

Kết quả này có thể dễ dàng chứng minh bằng quy nạp the0 $n$.

-----------

Quay trở lại bài toán, viết lại $an^3+bn^2+cn+d=A.\binom{n}{3}+B.\binom{n}{2}+C.\binom{n}{1}+D.\binom{n}{0}$. Do đa thức này nhận giá trị nguyên với mọi $n$ nguyên, thay lần lượt các giá trị của $n=1,2,3,4$ ta có :

$$C.\binom{1}{1}+D.\binom{1}{0}\in \mathbb{Z}$$

$$B.\binom{2}{2}+C.\binom{2}{1}+D.\binom{2}{0}\in \mathbb{Z}$$

$$A.\binom{3}{3}+B.\binom{3}{2}+C.\binom{3}{1}+D.\binom{3}{0}\in \mathbb{Z}$$

$$A.\binom{4}{3}+B.\binom{4}{2}+C.\binom{4}{1}+D.\binom{4}{0}\in \mathbb{Z}$$

Bằng 1 vài phép cộng trừ đơn giản ta sẽ có $A,B,C,D\in \mathbb{Z}$, từ đó :
$$6(an^3+bn^2+cn+d)=6\left(A.\binom{n}{3}+B.\binom{n}{2}+C.\binom{n}{1}+D.\binom{n}{0}\right)$$
$$=A.n(n-1)(n-2)+B.n(n-1)+C.n+D$$
Suy ra $a,b,c,d$ cũng là số nguyên.
***
Tổng quát :

Mọi đa thức bậc $n$ đều có thể được biểu diễn dưới dạng :

$$P_{n}(x)=b_0+b_1.\binom{x}{1}+b_2.\binom{n}{2}+.....+b_{n}.\binom{x}{n}$$

Và đa thức $P_{n}(x)$ nhận giá trị nguyên khi và chỉ khi tất cả các hệ số $b_i$ ($i=\overline{1;n}$) đều nguyên.

 

------------

Đề mới :

Bài 27 :

Tìm tất cả các số nguyên dương $a,b$ thoả mãn $ab>1$ và :

$$\frac{a^2+ab+b^2}{ab-1}\in \mathbb{N}$$

Bài 28 :

Ch0 $A$ là 1 số nguyên khác $0$, giải hệ phương trình nghiệm nguyên :

$$x+y^2+z^3=A\,,\,\frac{1}{x}+\frac{1}{y^2}+\frac{1}{z^3}=\frac{1}{A}\,,\,xy^2z^3=A^2$$


“There is no way home, home is the way.” - Thich Nhat Hanh

#39
nguyenthehoan

nguyenthehoan

    Sĩ quan

  • Thành viên
  • 392 Bài viết

Bài 28 :

Ch0 $A$ là 1 số nguyên khác $0$, giải hệ phương trình nghiệm nguyên :

$$x+y^2+z^3=A\,,\,\frac{1}{x}+\frac{1}{y^2}+\frac{1}{z^3}=\frac{1}{A}\,,\,xy^2z^3=A^2$$

 

Bài này làm sao ý!!!,thôi cứ giải vậy!

Theo bài ra ta có;$\frac{1}{x}+\frac{1}{y^{2}}+\frac{1}{z^{3}}=\frac{1}{x+y^{2}+z^{3}}$.

Đây là bài toán quen thuộc,từ đây suy ra:

$(x+y^{2})(y^{2}+z^{3})(z^{3}+x)=0$.

+)$x=-y^{2}$.Suy ra $z^{3}=A$ và $z^{3}.y^{4}=-A^{2}$.Do đó $y^{4}=-A$.Do đó 

$y^{4}=-z^{3}=-A$,Nên $y=k^{3},z=-k^{4}\Rightarrow A=-k^{12}$

+)Các trường hợp còn lại làm tương tự.



#40
WhjteShadow

WhjteShadow

    Thượng úy

  • Phó Quản lý Toán Ứng dụ
  • 1323 Bài viết

Lời giải bài 27:

Giả sử $\frac{a^2+b^2+ab}{ab-1}=n\in \mathbb{N}^{*}$, ta có :

$$a^2+b^2-(n-1).ab+n=0$$

Giả sử $(a_{0},b_{0})$ là nghiệm có tổng nhỏ nhất của phương trình trên, không mất tính tổng quát ta giả sử $a_0\geq b_0$

Ta xét phương trình :

$$f(a)=a^2-(n-1).b_0.a+b_0^2+n=0$$

The0 Vìete phương trình này ngoài nghiệm $a_0$ còn có nghiệm $a_1=(n-1).b_0-a_0=\frac{b_0^2+n}{a_0}$, suy ra $a_1\in \mathbb{N}^{*}$.

Vậy $(a_1;b_0)$ cũng là 1 cặp nghiệm của phương trình đầu. Nên $a_1+b_0\geq a_0+b_0\Rightarrow a_1\geq a_0\geq b_0$

The0 định lý về dấu của tam thức bậc 2, $f(b_0)\geq 0$

$$\Leftrightarrow b_0^2.(3-n)+n\geq 0$$

$$\Leftrightarrow b_0^2.(n-3)\leq n$$

Nhận thấy $n>3$ do $a^2+b^2\geq 2ab$ nên :

$$b_0\leq \frac{n}{n-3}\leq 4\Rightarrow n\in \{1;2\}$$

Nếu $b_0=1$, ta có $a^2-(n-1).a+(n+1)=0$, $\Delta =(n-1)^2-4(n+1)=(n-3)^2-12$ là số chính phương, nên $(n-2)^2-12=X^2\Leftrightarrow (n-3-X)(n-3+X)=12$, loại trực tiếp ta có $n=7$.

Nếu $b_0=2$, ta có $a^2-2(n-1).a+(n+4)=0$, $\Delta  =4(n-1)^2-4(n+4)=(2n-3)^2-21$ là số chính phương, nên $(2n-3)^2-21=Y^2\Leftrightarrow (2n-3-X)(2n-3+X)=9$, loại trực tiếp ta có $n=4$.

 ****

Đã x0ng công đoạn tìm $n$ với Vieta Jumping Menthod, h là tìm nghiệm với pt Markov.

$\bullet$ với $n=7$ ta có ngay nghiệm khởi đầu bằng $(1;2)$. (Với giả sử $a\leq b$)

Ta có phương trình :

$$a^2+b^2-6ab=-7$$

Với nhận xét : Nếu dãy số $\{u_n\}$ được xác định bởi $u_{0}=\alpha;u_{1}=\beta$, $u_{n+1}=a.u_n-u_{n-1}$ ta luôn có :

$$u_n^2+u_{n-1}^2-a.u_n.u_{n-1}= \alpha^2+\beta^2-a.\alpha.\beta$$

Ta dễ dàng có nghiệm của phương trình là $(a;b)=(u_{n};u_{n+1})$ với dãy $\{u_n\}$ được xác định bởi $u_{0}=1;u_{1}=4$, $u_{n+1}=6.u_n-u_{n-1}$

$\bullet$ với $n=4$ ta có ngay nghiệm khởi đầu bằng $(2;2)$. 

Phương trình trở thành $a^2+b^2-3ab=-4$.

Làm tương tự trên là có ngay nghiệm của phương trình là $(a;b)=(u_{n};u_{n+1})$ với dãy $\{u_n\}$ được xác định bởi $u_{0}=2;u_{1}=2$, $u_{n+1}=3.u_n-u_{n-1}$....

Spoiler

Bài toán 28 :

Ch0 số tự nhiên $n$, chứng minh rằng với mỗi $m=0;1;...;2^{n}-1$ luôn tồn tại $k$ sa0 ch0 :

$$\frac{1}{2}k(k-1)\equiv m \pmod{2^{n}}$$

Bài toán 29:

Ch0 $F_{n}$ là dãy số Fibonacci. Chứng minh rằng với mọi số tự nhiên $k$ luôn tồn tại số tự nhiên $i$ sa0 ch0 $10^{k}$ chia hết $F_i$

Bài toán 30 :

Ch0 $n>4$ là hợp số thỏa $n| \sigma (n). \varphi (n)+1$

Chứng minh rằng $\omega (n)\geq 3$ Với $\omega(n)$ là số các ước nguyên tố của $n$


Bài viết đã được chỉnh sửa nội dung bởi WhjteShadow: 01-07-2013 - 21:52

“There is no way home, home is the way.” - Thich Nhat Hanh





Được gắn nhãn với một hoặc nhiều trong số những từ khóa sau: tuyển tập-sưu tầm.

0 người đang xem chủ đề

0 thành viên, 0 khách, 0 thành viên ẩn danh